help with 57, please!!

Help With 57, Please!!

Answers

Answer 1

9514 1404 393

Answer:

  a) $554

  b) y = 138.50x

  c) see attached

Step-by-step explanation:

a) The cost is the product of the number of hours and the cost per hour.

  cost = (4 h)($138.50 /h) = $554.00 . . . . for a 4-hour course

__

b) As above, the cost (y) is the product of the number of hours (x) and the cost per hour ($138.50).

  y = 138.50x . . . . tuition for x credit hours.

__

c) As in part (b), the tuition is hours times 138.50. That means hours are tuition divided by 138.50.

Help With 57, Please!!

Related Questions

Adams Company revenues are $500 on invested capital of $250. Expenses are currently 60% of sales. If Angelo Company can reduce its capital investment by 20% in Adams Company, return on investment will be _____.

Answers

Answer:

100%

Step-by-step explanation:

The formula to calculate the return on investment is:

ROI=(Net Profit/Total Investment)*100

Net profit=Revenues-expenses=500-(500*0.6)=500-300=200

Total investment=250-(250*0.2)=250-50=200

Now, you can replace the values:

ROI= (200/200)*100

ROI= 100%

According to this, the answer is that If Angelo Company can reduce its capital investment by 20% in Adams Company, return on investment will be 100%.

1/5√75 -10√1/2+√125-2√1/2

Answers

Answer:6.91239069507

Step-by-step explanation:

HELP NOW - Please help me
100 POINTS

Answers

Answer:

SAS

43ft

Step-by-step explanation:

We know that two sides are equal

PQ = ST and QR = TU and the angle between them is equal Q = T

We can use the SAS (side angle side)

Since the triangles are congruent

PR = SU

6y+5 = 8y

Subtract 6y from each side

6y+5-6y = 8y-6y

5 = 2y

Divide by 2

5/2 = y

2.5 = y

PR = ST = 8y = 8(2.5) =20

The perimeter is 9+14+20 = 43

Answer:

B. =35ft

Step-by-step explanation:

The perimeter of a right angled triangle is a+b+c

which is 9+4+6y+5 =90

collect like terms

9+4+5+6y=90

18 + 6y = 90

6y = 90 - 18

6y= 72

divide both sides by 6

6y/6 = 72/6

y = 12

so therefore, the perimeter of ∆PQR is

9 + 12 + 14

= 35ft

ASAP!!! PLEASE help me solve this question! No nonsense answers, and solve with full solutions.

Answers

Answer:

Option (4)

Step-by-step explanation:

By the theorem of inscribed angles and the intercepted arc,

"In a circle, angles subtended by the same arc always measure the same and the arc measures the double of the inscribed angle."

If an inscribed angle in a circle measures 75° then all inscribed angles by the same arc will measure 75°.

In addition to this, measure of arc subtended by these inscribed angle will measure double of the inscribed angle (150°)

Therefore, Option (4) will be the answer.

PLEASE HELP, GIVING 55 POINTS IF YOU ANSWER (08.03/08.05 MC) A pair of equations is shown below: y = 3x − 5 y = 6x − 8 Part A: Explain how you will solve the pair of equations by substitution or elimination. Show all the steps and write the solution. (5 points) Part B: If the two equations are graphed, at what point will the lines representing the two equations intersect? Explain your answer. (5 points)

Answers

Answer:

  A. (1, -2)

  B. the lines intersect at the solution point: (1, -2).

Step-by-step explanation:

A. The equations can be solve by substitution by using the y-expression provided by one of them to substitute for y in the other.

This gives ...

  3x -5 = 6x -8

Adding 8-3x to both sides, we get ...

  3 = 3x

Dividing both sides by 3 gives ...

  1 = x

Substituting this value into the first equation, we can find y:

  y = 3(1) -5 = -2

The solution is (x, y) = (1, -2).

__

B. The lines intersect at the solution point, the point that satisfies both equations simultaneously. That point is (1, -2).

Find the value of x.

Answers

Answer:

[tex]\textsf{x=22.5}[/tex]

Step-by-step explanation:

[tex]\textsf{According to intersecting tangent - secant theorem,}[/tex]

[tex]\textsf{x=1/2[(4x+5)-50]}[/tex]

[tex]\textsf{2(x)=4x+5-50}[/tex]

[tex]\textsf{2x=4x-45}[/tex]

[tex]\textsf{2x-4x=-45}[/tex]

[tex]\textsf{-2x=-45}[/tex]

[tex]\textsf{x= -45/-2}[/tex]

[tex]\textsf{x=22.5}[/tex]

[tex]\textsf{OAmalOHopeO}[/tex]

A goat is grazing outside of a rectangular barn that has dimensions 20 ft by 30 ft.
He's tied to a corner of the barn with a 12 ft rope. Find the grazing area of the goat.

Answers

Answer:

??????????????????????????????

A graph of a system of equation with different slopes will have no solution ?

Answers

Answer:

Step-by-step explanation:

If we are talking slopes, we are talking linear equations, since lines are the only functions with slopes. If the slopes are the same, the lines will be parallel, and that is where we have a case of no solution because parallel lines will never intersect. NEVER EVER. If the slopes are different, eventually the lines will intersect somewhere.

Answer:

Step-by-step explanation:

False.  A system with different slopes is actually most likely to have a solution.

PLEASE HELP MEEE How can a company use a scatter plot to make future sale decisions

Answers

Answer:

by tracking data of how much money was made on one product in a certain amount of time

Step-by-step explanation:

Find the constant of proportionality (r) in the equation y = r x

Answers

Answer:

r = 11

Step-by-step explanation:

y = r x

r is the constant of proportionality

To find r pick any values of x and y provided and substitute it into the above formula and solve for r.

That's

using

x = 2

y = 22

We have

22 = 2r

Divide both sides by 2

r = 11

Therefore the constant of proportionality is 11

Hope this helps you

the work in an office takes 180 hours to complete every work
each person in the office works for 35 hours a week
what is the smallest number of people needed to complete the work?​

Answers

Answer:

Minimum People required = 5

Step-by-step explanation:

Total hours required to complete the work every week = 150 hrs.

Number of hours worked per week by one person = 32 hr

∴ Number of people required to complete the work per week = Total number of hrs to complete the work ÷ No of hrs work per person

∴ Number of people = 150 ÷ 32

∴ Number of people = 4.6875

This is the minimum number of people. But no of people cannot be a fraction.

Thus, rounding the number to next integer.

∴ Smallest number of people needed to complete the work = 5

A combination lock uses three numbers between 1 and 46 with​ repetition, and they must be selected in the correct sequence. Is the name of​ "combination lock"​ appropriate? Why or why​ not? Choose the correct answer below. A. ​No, because the multiplication counting rule would be used to determine the total number of combinations. B. ​Yes, because the combinations rule would be used to determine the total number of combinations. C. ​No, because factorials would be used to determine the total number of combinations. D. ​No, because the permutations rule would be used to determine the total number of combinations.

Answers

The correct answer is D. ​No because the permutations rule would be used to determine the total number of combinations.

Explanation:

The difference between a combination and a permutation is that in permutations the order is considered. This applies to the numbers in a lock because these need to be in order. Therefore, to analyze the permutations in a lock, the rule for permutations should be used. This includes the general formula P (n,r) =[tex]\frac{n!}{(n-r) !}[/tex]; in this, n is the number of objects and r refers to the objects used in a permutation. Thus, the term "combination" is inappropriate because this is a permutation, and the permutation rule should be used.

65. Given a segment with endpoints A and C and midpoint. If A(5, 8), and M(-3,2). Find the
location of C.

Answers

Answer:

C(-11,-4)

Step-by-step explanation:

Identify the rate of change and term 0
1. 3, 5, 7, 9, 11, 13, 15....​

Answers

Answer:

-1

mark me brainliest

Please give me the correct answer

Answers

Answer:

Height = 15

Step-by-step explanation:

[tex]Volume = 392.5\\r = 5\\h =?\\\\V= \frac{1}{3} \pi r^2 h\\\\392.5 = \frac{1}{3} \times 3.14 \times 5^2 \times h\\\\392.5 = \frac{78.5h}{3} \\\\392.5 = 26.16h\\\\\frac{392.5}{26.16} =\frac{26.16h}{26.16} \\\\h = 15.00[/tex]

Answer:

h=15 in

Step-by-step explanation:

V=πr²(h/3)

h=3v/πr²

h=[3(392.5)]/[3.14(5)²]

h= 15 in

A sandwich shop offers a choice of 5 types of bread, 4 types of meat, and 3 types of cheese. How many different sandwiches could be made with 1 type of bread, 1 type of meat, and 1 type of cheese?

Answers

Answer:

12 type of bread 1 type of meat, 1 type of cheese and 1 type of bread.☺️☺️

The Tama, Japan, monorail carries 92,700 riders

each day. If the monorail usually carries

5,150 riders per hour, how many hours does

the monorail run each day?

Answers

Answer:

The number of hours monorail run each day is 18.

Step-by-step explanation:

The total number of riders the monorail carry each day is:

N = 92700.

The number of riders the monorail carry per hour is:

n = 5150.

Compute the number of hours the monorail run each day as follows:

[tex]\text{Number of hours the monorail run each day}=\frac{N}{n}[/tex]

                                                                     [tex]=\frac{92700}{5150}\\\\=18[/tex]

Thus, the number of hours monorail run each day is 18.

Bryan decides he wants to help pay for a birthday party for his little brother at the ice rink. It cost $50 to rent the party room and then $4 for each person attending. Bryan only has $100 to spend at the party. a) What are the constraints for this situation? b) Find the domain and range for this situation. Make sure you include all values for each using correct notation.

Answers

Answer:

a) 4*x + 50 ≤ 100  

b) Domain   x (0 ; 12 )     Range   f(x)   ( 50  ; 98 )

Step-by-step explanation:

The constraint is:

4*x + 50 ≤ 100                   where "x" is the number of persons

b) Domain for x

x = 0    up to  x = 12           x (0 ; 12 )

c) Range for f(x)

f(x) = 4*x + 50

f(0)  = 4*0 + 50      f(0) =  50

f(12) = 4*12 + 50    f (12) =  98

f(x)   ( 50  ; 98 )

Three whole numbers have an HCF of 3 and an LCM of 180. Two of the numbers are 45 and 60. Find the third number.

Answers

Answer:

Step-by-step explanation:

45=3×3×5

60=2×2×3×5

L.C.M=180

2| 180

2|90

3|45

3|15

3|5

180=2×2×3×3×5

third number=2×3=6

or 2×2×3=12

or2×3×3=18

or 2×2×3×3=36

so third number can be one of  6,12,18,36

complete the first 4 steps for graphing the quadratic function given.
y= -x^2 -4x -3

ty<3

Answers

To be honest, I'm not sure which four steps your teacher is referring to. However, I'll show you one way to graph this.

A graph is simply a collection of points. Often those points are connected in some way (though they don't necessarily have to be) to form a curve.

Each point is of the form (x,y). To get each point, we pick random x values and determine their paired y value counterpart.

For example, if we pick x = -3, then,

y= -x^2 -4x -3

y= -(-3)^2 -4(-3) -3

y = -9 - 4(-3) - 3

y = -9 + 12 - 3

y = 0

This indicates that (-3, 0) is one point on the curve.

Let's repeat for x = -2

y= -x^2 -4x -3

y= -(-2)^2 -4(-2) -3

y = -4 - 4(-2) - 3

y = -4 + 8 - 3

y = 1

So (-2, 1) is another point on the curve.

Repeat this process as many times as you want. You should do at least 3 or 4 points in my opinion. The more points you generate, the more accurate the curve. After generating the points, you'll plot them all on the same xy grid. Then finally draw a curve through all of the points as shown below.

I used GeoGebra to make the graph.

help me with the picture please

Answers

Answer:  115

=====================================================

Explanation:

Refer to the diagram below. I've drawn diagonal that slopes upward. This diagonal cuts the quadrilateral into two triangles: One is equilateral and the other is isosceles.

The equilateral triangle marked in blue has all three angles 60 degrees each.

Note that the 60 and y angles combined to form 130, so,

60+y = 130

y = 130-60

y = 70

Then focus on the isosceles triangle (angles y, w and w). These three interior angles must add to 180

y+w+w = 180

70+2w = 180

2w = 180-70

2w = 110

w = 110/2

w = 55

This adds onto its adjacent neighbor of 60 to get w+60 = 55+60 = 115 degrees which is the value of x.

Evaluate the following expression.
28 – 10 – 15 = 3 =
and this is the order of operations

Answers

Answer:

28 - 10 - 15 - 3

=> 18 - 15 - 3

=> 3 - 3

=> 0

Another way:

=> 28 - 10 - 15 - 3

=> 28 - 25 - 3

=> 28 - 28

=> 0

The function f(x) = -(x - 3)2 + 9 can be used to represent the area of a rectangle with a perimeter of 12 units, as a
function of the length of the rectangle, x. What is the maximum area of the rectangle?
3 square units
6 square units
9 square units
12 square units

Answers

Answer:

  9 square units

Step-by-step explanation:

The function f(x) describes a parabola opening downward, with a vertex at (3, 9). The maximum value of f(x) is found at the vertex, where it is f(3) = 9.

The maximum area is 9 square units.

Answer:

9 c

Step-by-step explanation:

I need help please so if you could help that would be nice. Also i will make brainliest

Answers

No it is not a function. You have to be able to write y=mx+b

The Stem-and-Leaf Graph shows the amount of money each student spends on food per day in dollars. What is the median for the data in this Stem-and-Leaf Plot? A. $55 B. $73 C. $81 D. $84

Answers

Answer:

B) $73

Step-by-step explanation:

add all of your values and divide by the amount of values

52+55+55+55+59+64+66+68+72+73+73+73+73+75+81+81+83+84+84+86+87=

1,499

1,499 divided by 21 = 71.3809523...

which rounds to 73

HOPE THIS HELPS!!! :)

The median is of $48 in the stem leaf plot and option B is correct.

What is Statistics?

Statistics is the discipline that concerns the collection, organization, analysis, interpretation, and presentation of data.

The median of a data-set is the value that separates the bottom 50% from the upper 50% of values.

The graph has 16 values, already ordered.

It is an even number, hence the median is the mean of the 8th and the 9th values, which considering the key are both 48,

Hence, the median is of $48 and option B is correct.

To learn more on Statistics click:

https://brainly.com/question/30218856

#SPJ3

Stem-and-Leaf Plot shows the amount of money each student spends

on travel per day in dollars. What is the median for the data in this graph?

Stem Leaf

A) $35

B) $48

C) $53

D) $54

The sum of the digits of a two digit number is 10 when the dishes are reversed the number increases by 18 find the original number

Answers

Answer:

Step-by-step explanation:

Hello, we can write this number ab where a and b are integer betwen 0 and 9.

For instance, 54, a = 5, b = 4

And then, we can say ab = 10 * a + b.

For instance, 54  = 50 + 4 = 5*10 + 4.

The sum of the digits of a two digit number is 10.

a +  b = 10

When the dishes are reversed the number increases by 18.

10b + a = 18 + 10a + b

9b = 18 + 9a

b = 2 + a

We replace in the first equation to get.

a + 2 + a = 10

2a = 10 -2 = 8

a = 4

and then, b = 6

So, the number is 46.

Thank you

Pls pls help me! What is it

Answers

Answer:

H

Step-by-step explanation:

Jackie

[tex] \sqrt{9} + \sqrt{25} = 3 + 5 = 8[/tex]

Jesse

[tex] \sqrt{9 + 25} = \sqrt{34} [/tex]

Taking the difference, we get the answer to be

[tex]8 - \sqrt{34} [/tex]

46°
Х
>
X =
degrees
Hshshshsus

Answers

An exterior angle of a triangle is equal to the sum of its interior opposite angles.

⇛ x = 90° + 46°

⇛ x = 136°

Solve each system of equations 4x+6y=3 and -10x-15y=-4

Answers

Answer:

There is no solution

Step-by-step explanation:

they all subtract eachother out

(x^2+12x+36)-(x-1)^2
x^2-4xy+4y^2
16x^2-8x+1
x^2+4x+4
4x^2+12xy+9y^2
x^2-8x-16
4x^2-16
x^2-1
x^2+6x+9
9x^2-25y^4
16x^2-8x+1

Answers

Answer:

Hey I'm sorry I didn't get to answer your question it's just that I need the points because I don't have enough to get help with my question. I hope you get the answer that you need for you question. Good Luck :)

Step-by-step explanation:

Other Questions
In the course of selling a home to a buyer, a broker told the buyer that the home's foundation was "solid as a rock", when he knew for a fact that it was slowly sinking into the landfill on which it was built. In this situation, the broker's conduct would be BEST defined as a Is {3,} a defined set Why does the DNA that makes up the chromosomes need to replicate before the start of meiosis? Jonah will cover a cube in wrapping paper. Each edge of the cube is 25 cm long. What is the least amount ofwrapping paper he needs to cover the cube?15 625 square centimeters25 square centimeters37.5 square centimeters42 25 square centimetersSave and ExitNextSubmMO Tempe Office Services and Supplies (TOSS) provides various products and services in the Tempe Research Park, home to numerous high-tech and bio-tech companies. Making color copies is one of its most popular and profitable services. The controller performed a regression analysis of data from the Color Copy Department with the following results:Intercept 238.6R square 0.968Number of observation 6X coefficient 0.069Required:What is the variable cost per color copy for TOSS? PLEASE HELP ill give brainliest to best and correct answer QUESTION 3Individuals will experience positive or negative stress as a natural part of life.TrueFalse Which of the following is a major agricultural export of East Africa?Question 2 options:corncocoacoffeesugar cane The approach to measuring GDP that adds together compensation to employees, rents, interest income, dividends, undistributed corporate profits, proprietors' income, indirect business taxes, the consumption of fixed capital, and net foreign factor income earned in the United States is the: 19.The horses swim in the river to avoid danger. Which words make up the ENTIRE infinitive phrase in this sentence? Find the missing side. Round answer to the nearest tenth. Most reactions, including enzyme-catalyzed reactions, proceed faster at higher temperatures. However, for a given enzyme, the rate drops off abruptly at a certain temperature. Account for this behavior. Greek scientists and thinkers were able to make advances in science because they considered ___________ events the result of natural causes. unnatural Hellenistic phalanx natural A working group convened by NASAA has developed a model fee disclosure schedule to help investors better understand the costs involved in doing business with their broker-dealer. The template has broker-dealers disclose all of the following fees except:________. A) charges for late payments B) account inactivity fee C) issuance of a stock certificate D) advisory fees Given 12 consecutive integers, how many ways can three of these integers be selected to give a sum which divides by 4.Disclaimer: A lot of points to be given, Full explanation required. Not only answer. Remember the sum of the number must be divisible by 4. I think modular arithmetic is the way to solve it, but who knows??? state three effects ofcrop disease What is the primary source of energy in most living communities? What is the maximum most that can be extracted from 76 g og Cr2O3. A) 36 gB) 52 gC) 104 gD) 152 g What is the answer please What is the volume of a right square pyramid with a height of 3cm and a base that measures 8cm by 8cm?